Q6

 
andrea.feuer
Thanks Received: 0
Vinny Gambini
Vinny Gambini
 
Posts: 11
Joined: November 02nd, 2009
 
 
 

Q6

by andrea.feuer Thu Oct 14, 2010 12:12 am

Is the first question in this problem an orientation one? I am spending way too much time trying to figure this one out. What is the trick? I'm very lost. I am testing all the possible accurate matches and seem to be going in circles. Thank you for your help.
User avatar
 
ManhattanPrepLSAT1
Thanks Received: 1909
Atticus Finch
Atticus Finch
 
Posts: 2851
Joined: October 07th, 2009
 
 
 

Re: PT43, S4, Q6 - A Rowing Team

by ManhattanPrepLSAT1 Fri Oct 15, 2010 4:59 am

Great question. No, this question is not an orientation question. You cannot simply apply the rules on this one and eliminate each of the incorrect answers. The reason why is that this is not a complete and accurate ordering of the characters. Instead, this is a partial ordering.

Here, the best approach is to work your way from answer choice (A) through (E) and plug in each of the partial lists. You'll find that each one either explicitly breaks a constraint or implies conditions that would.

Sorry, no fast way to do this one, you simply have to work your way through them one by one...

Does that answer your question?
 
andrea.feuer
Thanks Received: 0
Vinny Gambini
Vinny Gambini
 
Posts: 11
Joined: November 02nd, 2009
 
 
 

Re: PT43, S4, Q6 - A Rowing Team

by andrea.feuer Fri Oct 15, 2010 10:02 pm

Thanks for your help and re-assurance that I wasn't going crazy in thinking this must be an orientation question. I did what you said and go to the right choice; it just takes a little longer than the typical orientation question. Thanks again.
 
adambrown07
Thanks Received: 0
Vinny Gambini
Vinny Gambini
 
Posts: 2
Joined: May 06th, 2011
 
 
 

Re: Q6

by adambrown07 Sat Sep 24, 2011 1:59 pm

Okay, I can't for the life of me see how C violates our diagram, and what makes it not a possible answer to Question 6. I have the diagram correct, and MSVZOL seems like it would meet the constraints of C, plus it wouldn't violate the diagram as far as I can tell. Can someone explain?
 
sergiok4
Thanks Received: 0
Vinny Gambini
Vinny Gambini
 
Posts: 2
Joined: January 24th, 2010
 
 
 

Re: Q6

by sergiok4 Tue Dec 06, 2011 10:23 pm

After reading your question I struggled with the answer (which I got correct while working on the section) until I read the question. The question asks which one could be an accurate matching so the answer is C since with the partial ordering that they gave in that answer the game board has to be this: LOZVSM (going 654321 - not sure why I oriented my diagram that way but now I can't really visualize it the other direction), which works for our diagram.
User avatar
 
ManhattanPrepLSAT1
Thanks Received: 1909
Atticus Finch
Atticus Finch
 
Posts: 2851
Joined: October 07th, 2009
 
 
 

Re: Q6

by ManhattanPrepLSAT1 Thu Dec 08, 2011 4:40 pm

Nice work Sergio! But I would highly recommend that you switch over to ordering

1 2 3 4 5 6

as opposed to

6 5 4 3 2 1

It'll make your life easier when you compare your work to the answer choices since it will be more likely to line up if they give you a hypothetical ordering.

adambrown07 Wrote:Okay, I can't for the life of me see how C violates our diagram, and what makes it not a possible answer to Question 6.


Just a note in case you were still confused - answer choice (C) is the correct answer!